Fixed typo in Young_and_Freedman_12/problem26.91.tex
[course.git] / latex / problems / Young_and_Freedman_12 / problem26.91.tex
1 \begin{problem*}{26.91}
2 As shown in Fig.~26.83, a network of resistors of resistances $R_1$
3 and $R_2$ extends to infinity toward the right.  Prove that the total
4 resistance $R_T$ of the infinite network is equal to
5 \begin{equation}
6   R_T = R_1 + \sqrt{R_1^2 + 2R_1R_2}
7 \end{equation}
8 (\emph{Hint:} Since the network is infinite, the resistance of the
9 network to the right of points $c$ and $d$ is also equal to $R_T$.)
10 \begin{center}
11 \begin{verbatim}
12     R1      R1      R1   
13 a-/\/\/-c-/\/\/-+-/\/\/-+-...
14         |       |       |
15         Z       Z       Z
16         Z R2    Z R2    Z R2
17         Z       Z       Z
18     R1  |   R1  |   R1  |
19 b-/\/\/-d-/\/\/-+-/\/\/-+-...
20 \end{verbatim}
21 \end{center}
22 \end{problem*}
23
24 \begin{solution}
25 Following the hint, we note that
26 \begin{align}
27   R_T &= R_1 + \p({\frac{1}{R_2} + \frac{1}{R_T}})^{-1} + R_1 \\
28   R_T - 2R_1 &= \p({\frac{1}{R_2} + \frac{1}{R_T}})^{-1} \\
29   (R_T - 2R_1) \cdot \p({\frac{1}{R_2} + \frac{1}{R_T}}) &= 1 \\
30   (R_T - 2R_1) \cdot (R_T + R_2) &= R_2R_T \\
31   R_T^2 - 2R_1R_T + R_2R_T - 2R_1R_2 &= R_2R_T \\
32   0 &= R_T^2 - 2R_1R_T - 2R_1R_2 \;.
33 \end{align}
34 Plugging this into the quadratic formula
35 \begin{equation}
36   R_T = \frac{2R_1 \pm \sqrt{4R_1^2 - 4\cdot1\cdot(-2R_1R_2)}}{2}
37     = R_1 \pm \sqrt{R_1^2 + 2R_1R_2}
38     = \ans{R_1 + \sqrt{R_1^2 + 2R_1R_2}} \;,
39 \end{equation}
40 which is what we set out to show.  Note that we chose the $+$ case
41 from $\pm$ because
42 \begin{equation}
43   R_1 < \sqrt{R_1^2 + 2R_1R_2} \;,
44 \end{equation}
45 and $R_T$ must be greater than zero.
46 \end{solution}